K
Khách

Hãy nhập câu hỏi của bạn vào đây, nếu là tài khoản VIP, bạn sẽ được ưu tiên trả lời.

AH
Akai Haruma
Giáo viên
17 tháng 11 2019

Lời giải:

Sửa đề: \(\frac{1}{(a+b+\sqrt{2(a+c)})^3}+\frac{1}{(b+c+\sqrt{2(b+a)})^3}+\frac{1}{(c+a+\sqrt{2(b+c)})^3}\leq \frac{8}{9}\)

--------------------------

Áp dụng BĐT AM-GM:

\(a+b+\sqrt{2(a+c)}=a+b+\sqrt{\frac{a+c}{2}}+\sqrt{\frac{a+c}{2}}\geq 3\sqrt[3]{\frac{(a+b)(a+c)}{2}}\)

\(\Rightarrow [a+b+\sqrt{2(a+c)}]^3\geq \frac{27}{2}(a+b)(a+c)\)

\(\Rightarrow \frac{1}{(a+b+\sqrt{2(a+c)})^3}\leq \frac{2}{27(a+b)(a+c)}\)

Hoàn toàn tương tự với các phân thức còn lại:

\(\Rightarrow \text{VT}\leq \frac{4(a+b+c)}{27(a+b)(b+c)(c+a)}(1)\)

Lại theo BĐT AM-GM:

\((a+b)(b+c)(c+a)=(a+b+c)(ab+bc+ac)-abc\geq (a+b+c)(ab+bc+ac)-\frac{(a+b+c)(ab+bc+ac)}{9}=\frac{8}{9}(a+b+c)(ab+bc+ac)(2)\)

Và:

\(16(a+b+c)\geq \frac{1}{a}+\frac{1}{b}+\frac{1}{c}=\frac{ab+bc+ac}{abc}\geq \frac{3(a+b+c)}{ab+bc+ac}\)

\(\Rightarrow ab+bc+ac\geq \frac{3}{16}(3)\)

Từ \((1);(2);(3)\Rightarrow \text{VT}\leq \frac{1}{6(ab+bc+ac)}\leq \frac{1}{6.\frac{3}{16}}=\frac{8}{9}\) (đpcm)

AH
Akai Haruma
Giáo viên
17 tháng 11 2019

Dấu "=" xảy ra khi $a=b=c=\frac{1}{4}$

28 tháng 8 2020

Áp dụng giả thiết và một đánh giá quen thuộc, ta được: \(16\left(a+b+c\right)\ge\frac{1}{a}+\frac{1}{b}+\frac{1}{c}=\frac{ab+bc+ca}{abc}=\frac{\left(ab+bc+ca\right)^2}{abc\left(ab+bc+ca\right)}\ge\frac{3\left(a+b+c\right)}{ab+bc+ca}\)hay \(\frac{1}{6\left(ab+bc+ca\right)}\le\frac{8}{9}\)

Đến đây, ta cần chứng minh \(\frac{1}{\left(a+b+\sqrt{2\left(a+c\right)}\right)^3}+\frac{1}{\left(b+c+\sqrt{2\left(b+a\right)}\right)^3}+\frac{1}{\left(c+a+\sqrt{2\left(c+b\right)}\right)^3}\le\frac{1}{6\left(ab+bc+ca\right)}\)

 Áp dụng bất đẳng thức Cauchy cho ba số dương ta có \(a+b+\sqrt{2\left(a+c\right)}=a+b+\sqrt{\frac{a+c}{2}}+\sqrt{\frac{a+c}{2}}\ge3\sqrt[3]{\frac{\left(a+b\right)\left(a+c\right)}{2}}\)hay \(\left(a+b+\sqrt{2\left(a+c\right)}\right)^3\ge\frac{27\left(a+b\right)\left(a+c\right)}{2}\Leftrightarrow\frac{1}{\left(a+b+2\sqrt{a+c}\right)^3}\le\frac{2}{27\left(a+b\right)\left(a+c\right)}\)

Hoàn toàn tương tự ta có \(\frac{1}{\left(b+c+2\sqrt{b+a}\right)^3}\le\frac{2}{27\left(b+c\right)\left(b+a\right)}\)\(\frac{1}{\left(c+a+2\sqrt{c+b}\right)^3}\le\frac{2}{27\left(c+a\right)\left(c+b\right)}\)

Cộng theo vế các bất đẳng thức trên ta được \(\frac{1}{\left(a+b+\sqrt{2\left(a+c\right)}\right)^3}+\frac{1}{\left(b+c+\sqrt{2\left(b+a\right)}\right)^3}+\frac{1}{\left(c+a+\sqrt{2\left(c+b\right)}\right)^3}\le\frac{4\left(a+b+c\right)}{27\left(a+b\right)\left(b+c\right)\left(c+a\right)}\)Phép chứng minh sẽ hoàn tất nếu ta chỉ ra được \(\frac{4\left(a+b+c\right)}{27\left(a+b\right)\left(b+c\right)\left(c+a\right)}\le\frac{1}{6\left(ab+bc+ca\right)}\)\(\Leftrightarrow\left(a+b\right)\left(b+c\right)\left(c+a\right)\ge\frac{8}{9}\left(ab+bc+ca\right)\left(a+b+c\right)\)

Đây là một đánh giá đúng, thật vậy: đặt a + b + c = p; ab + bc + ca = q; abc = r thì bất đẳng thức trên trở thành \(pq-r\ge\frac{8}{9}pq\Leftrightarrow\frac{1}{9}pq\ge r\)*đúng vì \(a+b+c\ge3\sqrt[3]{abc}\)\(ab+bc+ca\ge3\sqrt[3]{\left(abc\right)^2}\))

Vậy bất đẳng thức được chứng minh

Đẳng thức xảy ra khi \(a=b=c=\frac{1}{4}\)

2 tháng 12 2020

Đặt \(\left(\frac{1}{a},\frac{1}{b},\frac{1}{c}\right)=\left(x,y,z\right)\)

\(x+y+z\ge\frac{x^2+2xy}{2x+y}+\frac{y^2+2yz}{2y+z}+\frac{z^2+2zx}{2z+x}\)

\(\Leftrightarrow x+y+z\ge\frac{3xy}{2x+y}+\frac{3yz}{2y+z}+\frac{3zx}{2z+x}\)

\(\frac{3xy}{2x+y}\le\frac{3}{9}xy\left(\frac{1}{x}+\frac{1}{x}+\frac{1}{y}\right)=\frac{1}{3}\left(x+2y\right)\)

\(\Rightarrow\Sigma_{cyc}\frac{3xy}{2x+y}\le\frac{1}{3}\left[\left(x+2y\right)+\left(y+2z\right)+\left(z+2x\right)\right]=x+y+z\)

Dấu "=" xảy ra khi x=y=z

23 tháng 3 2017

mình chịu bó tay

24 tháng 3 2017

sao lại thế -.-

1 tháng 8 2020

Xét \(\frac{a^3}{a^2+ab+b^2}-\frac{b^3}{a^2+ab+b^2}=\frac{\left(a-b\right)\left(a^2+ab+b^2\right)}{a^2+ab+b^2}=a-b\)

Tương tự, ta được: \(\frac{b^3}{b^2+bc+c^2}-\frac{c^3}{b^2+bc+c^2}=b-c\)\(\frac{c^3}{c^2+ca+a^2}-\frac{a^3}{c^2+ca+a^2}=c-a\)

Cộng theo vế của 3 đẳng thức trên, ta được: \(\left(\frac{a^3}{a^2+ab+b^2}+\frac{b^3}{b^2+bc+c^2}+\frac{c^3}{c^2+ca+a^2}\right)\)\(-\left(\frac{b^3}{a^2+ab+b^2}+\frac{c^3}{b^2+bc+c^2}+\frac{a^3}{c^2+ca+a^2}\right)=0\)

\(\Rightarrow\frac{a^3}{a^2+ab+b^2}+\frac{b^3}{b^2+bc+c^2}+\frac{c^3}{c^2+ca+a^2}\)\(=\frac{b^3}{a^2+ab+b^2}+\frac{c^3}{b^2+bc+c^2}+\frac{a^3}{c^2+ca+a^2}\)

Ta đi chứng minh BĐT phụ sau: \(a^2-ab+b^2\ge\frac{1}{3}\left(a^2+ab+b^2\right)\)(*)

Thật vậy: (*)\(\Leftrightarrow\frac{2}{3}\left(a-b\right)^2\ge0\)*đúng*

\(\Rightarrow2LHS=\Sigma_{cyc}\frac{a^3+b^3}{a^2+ab+b^2}=\Sigma_{cyc}\text{ }\frac{\left(a+b\right)\left(a^2-ab+b^2\right)}{a^2+ab+b^2}\)\(\ge\Sigma_{cyc}\text{ }\frac{\frac{1}{3}\left(a+b\right)\left(a^2+ab+b^2\right)}{a^2+ab+b^2}=\frac{1}{3}\text{​​}\Sigma_{cyc}\left[\left(a+b\right)\right]=\frac{2\left(a+b+c\right)}{3}\)

\(\Rightarrow LHS\ge\frac{a+b+c}{3}=RHS\)(Q.E.D)

Đẳng thức xảy ra khi a = b = c

P/S: Có thể dùng BĐT phụ ở câu 3a để chứng minhxD:

27 tháng 7 2020

1) ta chứng minh được \(\Sigma\frac{a^4}{\left(a+b\right)\left(a^2+b^2\right)}=\Sigma\frac{b^4}{\left(a+b\right)\left(a^2+b^2\right)}\)

\(VT=\frac{1}{2}\Sigma\frac{a^4+b^4}{\left(a+b\right)\left(a^2+b^2\right)}\ge\frac{1}{4}\Sigma\frac{a^2+b^2}{a+b}\ge\frac{1}{8}\Sigma\left(a+b\right)=\frac{a+b+c+d}{4}\)

bài 2 xem có ghi nhầm ko

11 tháng 4 2017

\(\frac{a^3}{\left(1+b\right)\left(1+c\right)}+\frac{b^3}{\left(1+c\right)\left(1+a\right)}+\frac{c^3}{\left(1+a\right)\left(1+b\right)}\)

Ta có:

\(\frac{a^3}{\left(1+b\right)\left(1+c\right)}+\frac{1+b}{8}+\frac{1+c}{8}\ge\frac{3a}{4}\)

\(\Leftrightarrow\frac{a^3}{\left(1+b\right)\left(1+c\right)}\ge\frac{6a-b-c-2}{8}\)

Tương tự ta có: \(\hept{\begin{cases}\frac{b^3}{\left(1+c\right)\left(1+a\right)}\ge\frac{6b-c-a-2}{8}\\\frac{c^3}{\left(1+a\right)\left(1+b\right)}\ge\frac{6c-a-b-2}{8}\end{cases}}\)

Cộng vế theo vế ta được

\(\frac{a^3}{\left(1+b\right)\left(1+c\right)}+\frac{b^3}{\left(1+c\right)\left(1+a\right)}+\frac{c^3}{\left(1+a\right)\left(1+b\right)}\ge\frac{6a-b-c-2}{8}+\frac{6b-c-a-2}{8}+\frac{6c-a-b-2}{8}\)

\(=\frac{a+b+c}{2}-\frac{3}{4}\ge\frac{3}{2}.\sqrt[3]{abc}-\frac{3}{4}=\frac{3}{2}-\frac{3}{4}=\frac{3}{4}\)

10 tháng 4 2017

Mai mình làm cho

9 tháng 7 2017

Lần sau đăng ít 1 thôi đăng nhiều ngại làm, bn đăng nhiều nên tui hướng dẫn sơ qua thôi tự làm đầy đủ vào vở

Bài 1:

Áp dụng BĐT AM-GM ta có:

\(a^4+b^4\ge2a^2b^2;b^4+c^4\ge2b^2c^2;c^4+a^4\ge2c^2a^2\)

Cộng theo vế 3 BĐT trên rồi thu gọn

\(a^4+b^4+c^4\ge a^2b^2+b^2c^2+c^2a^2\)

Áp dụng tiếp BĐT AM-GM

\(a^2b^2+b^2c^2=b^2\left(a^2+c^2\right)\ge2b^2ac\)

Tương tự rồi cộng theo vế có ĐPCM

Bài 2:

Quy đồng  BĐT trên ta có:

\(\frac{a^2}{b^2}+\frac{b^2}{a^2}-\frac{a}{b}-\frac{b}{a}\ge0\)

\(\Leftrightarrow\frac{\left(a-b\right)^2\left(a^2+ab+b^2\right)}{a^2b^2}\ge0\) (luôn đúng)

Bài 4: Áp dụng BĐT AM-GM 

\(a^3+b^3=\left(a+b\right)\left(a^2-ab+b^2\right)\)

\(\ge\left(a+b\right)\left(2ab-ab\right)=ab\left(a+b\right)\)

\(\Rightarrow\frac{a^3+b^3}{ab}\ge\frac{ab\left(a+b\right)}{ab}=a+b\)

Tương tự rồi cộng theo vế

Bài 5: sai đề tự nhien có dấu - :v nghĩ là +

9 tháng 7 2017

ai k mình k lại [ chỉ 3 người đầu tiên mà trên 10 điểm hỏi đáp ]